Đến nội dung

le_hoang1995 nội dung

Có 342 mục bởi le_hoang1995 (Tìm giới hạn từ 09-06-2020)



Sắp theo                Sắp xếp  

#332023 $P=\dfrac{x^3y^3}{(x+yz)(y+zx)(z+xy)^2}$

Đã gửi bởi le_hoang1995 on 05-07-2012 - 01:36 trong Bất đẳng thức và cực trị

Bài 21: Cho 3 số thực $a,b,c>0$ thỏa $a+b+1=c$. Tìm GTLN của biểu thức $$T=\frac{a^3b^3}{(a+bc)(b+ac)(c+ab)^2}$$

Thế c vào biểu thức, ta được:
Với a,b dương, tìm max:
$$T=\frac{a^3b^3}{(a+b)^2(a+1)^3(b+1)^3}$$
Theo BĐT AM-GM và Holder, ta có:
$$T=\frac{a^3b^3}{(a+b)^2(a+1)^3(b+1)^3}\leq \frac{a^3b^3}{4ab.(\sqrt[6]{a^3b^3}+1)^6}=\frac{a^2b^2}{4(\sqrt{ab}+1)^6}=\frac{x^4}{4(x+1)^6}$$
Khảo sát hàm số trên , ta được $T\leq \frac{4}{3^6}$ khi $x=2 \rightarrow a=b=2$.




#332102 Với mọi $x_{1},x_{2},...x_{n}\in R;a_{1},a_{2},...a_{n}>0$...

Đã gửi bởi le_hoang1995 on 05-07-2012 - 12:44 trong Bất đẳng thức - Cực trị

Bài toán: Với mọi $x_{1},x_{2},...x_{n}\in R;a_{1},a_{2},...a_{n}>0$ hãy chứng minh bất đẳng thức sau:

$\frac{1}{n-1}\left ( x_{1}+x_{2}+...+x_{n} \right )^2\leq \left ( \frac{a_{1}^{n+1}}{S-a_{1}} +...+\frac{a_{n}^{m+1}}{S-a_{n}}\right )\left ( \frac{x_{1}^{2}}{a_{1}^{m}}+ \frac{x_{2}^{2}}{a_{2}^{m}}+...+\frac{x_{n}^{2}}{a_{n}^{m}}\right )$
Trong đó: $m,n \in N;m\geq 2$ và $S=a_1+a_2+...+a_n$




P/s: Bác nào sửa giùm e cái tiêu đề

Cái số mũ đầu tiên của VP có vấn đề rồi, giải như sau. Theo Chebusep 2 lần và CS ta có
$$\frac{a_1^{m+1}}{S-a_1}+...+\frac{a_n^{m+1}}{S-a_n}\geq \frac{1}{n}.(a_1^{m+1}+...+a_n^{m+1}).\left ( \frac{1}{S-a_1}+...+\frac{1}{S-a_n} \right )$$
$$\geq \frac{1}{n}.\frac{1}{n}.(a_1+...+a_n).(a_1^m+...+a_n^m).\frac{n^2}{\sum_{k=1}^{n} (S-a_k)}$$
$$=(a_1+...+a_n).(a_1^m+...+a_n^m).\frac{1}{(n-1).S}=\frac{a_1^m+...+a_n^m}{n-1}$$
Như vậy áp dụng CS ta được $$VP\geq \frac{a_1^m+...+a_n^m}{n-1}.\frac{(x_1+x_2+...+x_n)^2}{a_1^m+...+a_n^m}=VT$$
Dấu bằng xảy ra khi $a_1=a_2=...=a_n$ và $x_1=x_2=...=x_n$



#329015 Chứng minh rằng: $\left | \frac{a_{2}a_{1}}{a_{3}a_{0}} \...

Đã gửi bởi le_hoang1995 on 25-06-2012 - 15:37 trong Bất đẳng thức và cực trị

Cho đa thức P(x)=$a_{3}x^{3}+a_{2}x^{2}+a_{1}x+a_{0}$ với $a_{3}\neq 0,a_{0}\neq 0$
biết rằng đa thức có ba nghiệm dương.Chứng minh rằng:
$\left | \frac{a_{2}a_{1}}{a_{3}a_{0}} \right |\geq 9$

Vì đa thức có 3 nghiệm dương nên có thể viết nó dưới dạng
$$P(x)=a_3.(x-x_1)(x-x_2)(x-x_3)$$ với $x_1,x_2,x_3>0$
Nhân ra ta có:$P(x)= a_3.x^3-a_3.(x_1+x_2+x_3).x^2+a_3(x_1x_2+x_2x_3+x_3x_1).x-a_3.x_1.x_2.x_3$
Đồng quy hệ số ta được
$$\left\{\begin{matrix}
a_2=-a_3.(x_1+x_2+x_3)\\a_1=a_3.(x_1x_2+x_2x_3+x_3x_1)
\\ a_0=-a_3.x_1.x_2.x_3

\end{matrix}\right.$$
Thay vào biểu thức, đổi các nghiệm tương ứng là a,b,c cho dễ nhìn nhé, ta phải chứng minh
$$\left | \frac{-(a+b+c)(ab+bc+ca)}{abc} \right |\geq 9$$
$$\Leftrightarrow \frac{(a+b+c)(ab+bc+ca)}{abc}\geq 9$$
Vì $a,b,c>0$. Nhưng đây chính là BĐT AM-GM
$$(a+b+c)(ab+bc+ca)\geq 3\sqrt[3]{abc}.3\sqrt[3]{(abc)^2}=9abc$$
ĐPCM. Dấu bằng xảy ra khi $a=b=c$ hay đa thức $P(x)$ có 3 nghiệm bằng nhau và khác 0



#329059 Chứng minh rằng: $\left | \frac{a_{2}a_{1}}{a_{3}a_{0}} \...

Đã gửi bởi le_hoang1995 on 25-06-2012 - 17:46 trong Bất đẳng thức và cực trị

bạn đồng nhất hệ thức bị sai rồi, theo mình là $$\left\{\begin{matrix}
a_2=-a_3.(x_1+x_2+x_3)\\a_1=a_3.(x_1x_2+x_2x_3+x_3x_1)
\\ a_0=-a_3.x_1.x_2.x_3
\end{matrix}\right.$$

Ừ, mình gõ nhầm, tại không để ý kiểm tra lại kĩ :lol:



#336642 Chứng minh: $ax+by+cz+\sqrt{(a^2+b^2+c^2)(x^2+y^2+z^2)}...

Đã gửi bởi le_hoang1995 on 16-07-2012 - 22:57 trong Bất đẳng thức và cực trị

Với $a,b,c,x,y,z\in R$. Chứng minh: $$ax+by+cz+\sqrt{(a^2+b^2+c^2)(x^2+y^2+z^2)}\ge \frac{2}{3}(a+b+c)(x+y+z)$$

Một cách giải mà mình đọc được ( ở đâu không nhớ)
Ta có đẳng thức $$(2x+2y-z)^2+(2y+2z-x)^2+(2z+2x-y)^2=9(x^2+y^2+z^2)$$.
$$c(2x+2y-z)+a(2y+2z-x)+b(2z+2x-y)=2(x+y+z)(a+b+c)-3(ax+by+cz)$$.
Áp dụng BĐT CS, ta được
$$[2(x+y+z)(a+b+c)-3(ax+by+cz)]^2=[c(2x+2y-z)+a(2y+2z-x)+b(2z+2x-y)]^2$$
$$\leq (a^2+b^2+c^2).\left [ (2x+2y-z)^2+(2y+2z-x)^2+(2z+2x-y)^2 \right ]=9(a^2+b^2+c^2)(x^2+y^2+z^2)$$
$$\Leftrightarrow 2(x+y+z)(a+b+c)-3(ax+by+cz)\leq 3\sqrt{(a^2+b^2+c^2)(x^2+y^2+z^2)}$$
$$\Leftrightarrow ax+by+cz+\sqrt{(a^2+b^2+c^2)(x^2+y^2+z^2)}\ge \frac{2}{3}(a+b+c)(x+y+z)$$



#297201 Cho a,b,c>0. Chứng minh $P\geq\frac{a+b+c}{3}$

Đã gửi bởi le_hoang1995 on 29-01-2012 - 10:54 trong Bất đẳng thức và cực trị

Theo BĐT côsi ta có:

$\frac{a^3}{a^2+ab+b^2}=a-\frac{ab(a+b)}{a^2+ab+b^2}\geq a-\frac{ab(a+b)}{3\sqrt[3]{a^3b^3}}=a-\frac{a+b}{3}$

Tương tự 2 BĐT nữa rồi cộng lại ta có ĐPCM

Dấu bằng xảy ra khi $a=b=c$



#332083 Tìm giá trị lớn nhất của biểu thức

Đã gửi bởi le_hoang1995 on 05-07-2012 - 10:31 trong Bất đẳng thức và cực trị

Ta sẽ chứng minh $\sqrt{\frac{1+x^2}{2}}\leq x+1-\sqrt{x}$
$$\Leftrightarrow 1+x^2\leq 2(x-\sqrt{x}+1)^2\Leftrightarrow 1+x^2\leq 2(x^2+x+1-2x\sqrt{x}+2x-2\sqrt{x})$$
$$\Leftrightarrow x^2-4x\sqrt{x}+6x-4\sqrt{x}+1\geq 0$$
$$\Leftrightarrow (\sqrt{x}-1)^4\geq 0$$
Đúng, suy ra $\sqrt{1+x^2}\leq \sqrt{2}(x+1)-\sqrt{2x}$. Lại theo CS

$3\geq \sqrt{3(x+y+z)}\geq \sqrt{x}+\sqrt{y}+\sqrt{z}$. Thay vào bài, ta được

$VT\leq \sqrt{2}(x+y+z+3)+(3-\sqrt{2}).(\sqrt{x}+\sqrt{y}+\sqrt{z})\leq \sqrt{2}.6+(3-\sqrt{2}).3$
$=9+3\sqrt{2}$
Dấu bằng xảy ra khi $x=y=z=1$




#329067 CMR: $\sum_{i=2}^{n}\frac{i-1}{T_i}<1$

Đã gửi bởi le_hoang1995 on 25-06-2012 - 18:14 trong Bất đẳng thức và cực trị

Đặt: $T_i=1.2.3...i.$ CMR: $\sum_{i=2}^{n}\frac{i-1}{T_i}<1$

Cái $T_i=1.2.3...i.$ người ta gọi là i giai thừa và kí hiệu là $i!$ bạn ạ. Ta có
$$\frac{i-1}{i!}=\frac{1}{(i-1)!}-\frac{1}{i!}$$
$$\sum_{i=2}^{n}\frac{i-1}{i!}=\frac{1}{2!}+\frac{2}{3!}+...+\frac{n-1}{n!}=\frac{1}{2}+\frac{1}{2!}-\frac{1}{3!}+\frac{1}{3!}-\frac{1}{4!}+...+\frac{1}{(n-1)!}-\frac{1}{n!}$$
$$=1-\frac{1}{n!}<1$$



#293943 Cho $x_{0}=2009$ $x_{n}=\frac{-2009}{n}\sum_...

Đã gửi bởi le_hoang1995 on 15-01-2012 - 10:07 trong Dãy số - Giới hạn

Mời mọi người làm bài sau, khá hay:

Cho $x_{0}=2009$
$x_{n}=\frac{-2009}{n}\sum_{k=0}^{n-1}x_{k}$


Tính tổng $S=\sum_{0}^{2009}(2^{k}*x_{k})$

Cảm ơn anh Xusinst, em đã sửa lại đề rùi



#535168 Tìm GTLN và GTNN của A = x - y.

Đã gửi bởi le_hoang1995 on 28-11-2014 - 13:49 trong Bất đẳng thức và cực trị

Biết rằng $x^{2}+y^{2}=x+y$. Tìm GTLN và GTNN của A = x - y.

 

Đặt $x+y=a$ ta có: $$a=x+y=x^2+y^2=(x+y)^2-2xy \Rightarrow 2xy=a^2-a$$

$$A^2=(x-y)^2=(x+y)^2-4xy=a^2-2.(a^2-a)=-a^2+2a=1-(a-1)^2\leq 1$$

Suy ra $-1\leq A\leq 1$

Đẳng thức xảy ra tại $a=1$ nên ta giải hệ $\left\{\begin{matrix}
x+y=1\\x^2+y^2=1

\end{matrix}\right.$

Giải hệ trên, ta thu được nghiệm

Vậy $Max_{A}=1$ khi $ x=1 $ và$ y=0$
$Min_{A}=-1$ khi $x=0$ và $y=1$




#326667 CMR: $\sum (1+b+c-bc)cosA \leq 3$

Đã gửi bởi le_hoang1995 on 18-06-2012 - 13:50 trong Bất đẳng thức và cực trị

Góp cho topic 1 bài vui.

Bài 120: Trong tam giác ABC, CMR:

$(1+a+b-ab)cosC+(1+b+c-bc)cosA+(1+c+a-ca)cosB\leq 3$


Ta có VT của BĐT cần chứng minh rằng:
$\left( {\cos A + \cos B + \cos C} \right) + \left( {b + c} \right)\cos A + \left( {c + a} \right)\cos B + \left( {a + b} \right)\cos C - \left( {ab\cos C + bc\cos A + ca\cos B} \right)$
Đặt:
$\begin{array}{l}
P = \cos A + \cos B + \cos C\\
Q = \left( {b + c} \right)\cos A + \left( {c + a} \right)\cos B + \left( {a + b} \right)\cos C\\
S = ab\cos C + bc\cos A + ca\cos B
\end{array}$
Dễ thấy: $P \le \frac{3}{2}$
Mặt khác ta có:
$b\cos C + c\cos B = 2R\left( {\sin B\cos C + \sin C\cos B} \right) = 2R\sin \left( {B + C} \right) = 2R\sin A = a$
Tương tự:
$\begin{array}{l}
c\cos A + a\cos C = b\\
a\cos B + b\cos A = c
\end{array}$
$ \Rightarrow Q = a + b + c$
Lại có:
$ab\cos C + bc\cos A + ca\cos B = \frac{{{a^2} + {b^2} - {c^2}}}{2} + \frac{{{b^2} + {c^2} - {a^2}}}{2} + \frac{{{c^2} + {a^2} - {b^2}}}{2}$
$ \Rightarrow S = \frac{{{a^2} + {b^2} + {c^2}}}{2}$
$P + Q + R \le \frac{3}{2} + a + b + c - \frac{{{a^2} + {b^2} + {c^2}}}{2} = 3 - \frac{{{{\left( {a - 1} \right)}^2} + {{\left( {b - 1} \right)}^2} + {{\left( {c - 1} \right)}^2}}}{3} \le 3$
Vậy ta có đpcm.


Bài này còn 1 cách khác là sử dụng chebusep

Giả sử $a\geq b\geq c\Rightarrow A\geq B\geq C\Rightarrow cosA\leq cosB\leq cosC$

$\sum (1+a+b)cosC-\sum abcosC\leq \frac{1}{3}(3+2(a+b+c))(cosA+cosB+cosC)-\frac{a^2+b^2+c^2}{2}$

$\leq \frac{1}{3}(3+2(a+b+c))*\frac{3}{2}-\frac{a^2+b^2+c^2}{2}=(a+b+c)+\frac{3}{2} -\frac{a^2+b^2+c^2}{2}$

Ta cần chứng minh $\frac{a^2+b^2+c^2+3}{2}\geq a+b+c$

Nhưng nó đúng vì $\frac{a^2+b^2+c^2+3}{2}\geq \sqrt{3(a^2+b^2+c^2)}\geq a+b+c$




#335092 $\sum {\frac{1}{{{{\left( {a + b + \sqrt {2a + 2c} }...

Đã gửi bởi le_hoang1995 on 13-07-2012 - 00:37 trong Bất đẳng thức và cực trị

Cho a, b, c là các số thực dương thõa mãn $\frac{1}{a} + \frac{1}{b} + \frac{1}{c} \le 16\left( {a + b + c} \right)$. Chứng minh rằng:
\[\frac{1}{{{{\left( {a + b + \sqrt {2a + 2c} } \right)}^3}}} + \frac{1}{{{{\left( {b + c + \sqrt {2b + 2a} } \right)}^3}}} + \frac{1}{{{{\left( {c + a + \sqrt {2c + 2b} } \right)}^3}}} \le \frac{8}{9}\]

Từ giả thiết, ta có $ab+bc+ca\leq 16abc(a+b+c)\leq \frac{16(ab+bc+ca)^2}{3}\Rightarrow ab+bc+ca\geq \frac{3}{16}$
Áp dụng BĐT AM-GM ta được $$(a+b+\sqrt{2(a+c)})^3=(a+b+\sqrt{\frac{a+c}{2}}+\sqrt{\frac{a+c}{2}})^3\geq \frac{27(a+b)(a+c)}{2}$$
$$\Rightarrow VT\leq \sum \frac{2}{27(a+b)(a+c)}=\frac{4}{27}. \frac{a+b+c}{(a+b)(b+c)(c+a)}$$
Sử dụng BĐT quen thuộc $\frac{(a+b)(b+c)(c+a)}{8}\geq \frac{(a+b+c)(ab+bc+ca)}{9}$
( Chứng minh: nó tương với $a(b-c)^2+b(c-a)^2+c(a-b)^2\geq 0$ đúng)
Ta được $$\frac{(a+b)(b+c)(c+a)}{8}\geq \frac{(a+b+c)(ab+bc+ca)}{9}\geq \frac{(a+b+c).3}{9.16}=\frac{a+b+c}{48}$$
$$\Rightarrow \frac{a+b+c}{(a+b)(b+c)(c+a)}\leq 6$$
Thay vào trên $$VT\leq \frac{4(a+b+c)}{27(a+b)(b+c)(c+a)}\leq \frac{4.6}{27}=\frac{8}{9}$$
Dấu bằng xảy ra khi $a=b=c=\frac{1}{4}$



#535175 Chứng minh $2\sum a^{2014}\geq \sum a^{201...

Đã gửi bởi le_hoang1995 on 28-11-2014 - 14:19 trong Bất đẳng thức và cực trị

Cho $a; b; c$ là các số thực dương. Chứng minh:

$2(a^{2014}+b^{2014}+c^{2014})\geq a^{2013}(b+c)+b^{2013}(a+c)+c^{2013}(a+b)$

Áp dụng BĐT Cosi cho 2014 số, ta được

$$2013.a^{2014}+b^{2014}\geq 2014.\sqrt[2014]{a^{2013.2014}.b^{2014}}=2014.a^{2013}.b$$

Tương tự, ta được $$2013.a^{2014}+c^{2014}\geq 2014.\sqrt[2014]{a^{2013.2014}.c^{2014}}=2014.a^{2013}.c$$

Cộng lại ta được $$2.2013.a^{2014}+b^{2014}+c^{2014}\geq 2014.a^{2013}.(b+c)$$

Hoàn toàn tương tự, thêm hai BĐT nữa, cộng lại, ta được ĐPCM




#315843 $12x^{2}+2y^{2}=5$ chứng minh rằng: $x+y+\frac{1}{xy}...

Đã gửi bởi le_hoang1995 on 11-05-2012 - 17:30 trong Bất đẳng thức và cực trị

Áp dụng BĐT AM-GM cho 7 số

$VT=x+\frac{y}{2}+\frac{y}{2}+4.\frac{1}{4xy}\geq 7\sqrt[7]{\frac{xy^2}{2^{10}(xy)^4}}=\frac{7}{\sqrt[7]{2^{10}x^3y^2}}$

Ta cần chứng minh

$\sqrt[7]{2^{10}x^3y^2}\leq 2\Leftrightarrow 2^3x^3y^2\leq 1$

$\Leftrightarrow 2^6y^4x^6\leq 1\Leftrightarrow 2^4x^6(5-12x^2)^2\leq 1$

Thật vậy theo AM-GM

$8x^2.8x^2.8x^2.(5-12x^2).(5-12x^2)\leq \left ( \frac{3.8x^2+10-24x^2}{5} \right )^5=2^5$

$\Leftrightarrow 2^4x^6(5-12x^2)^2\leq 1$

ĐPCM. Dấu bằng xảy ra khi $x=\frac{1}{2};y=1$



#335235 Cho $a\ge b\ge c>0$. Chứng minh rằng : $$...

Đã gửi bởi le_hoang1995 on 13-07-2012 - 16:04 trong Bất đẳng thức và cực trị

Bài toán [Gần với bài VMO]
Cho $a\ge b\ge c>0$. Chứng minh rằng :
$$\dfrac{a^2b}{c^2}+\dfrac{b^2c}{a^2}+\dfrac{c^2a}{b^2}\ge a+b+c$$

Không biết có được gọi là cách 2 không, chờ mọi người kiểm chứng giùm nhé.
Trừ vế trực tiếp, ta cần chứng minh:
$$a^4b^3+b^4c^3+c^4a^3-a^2b^2c^2(a+b+c) \ge 0$$
Ta phân tích
$$a^4b^3+b^4c^3+c^4a^3-a^2b^2c^2(a+b+c)=a^3b^2(ab-c^2)+b^3c^2(bc-a^2)+c^3a^2(ca-b^2)$$
$$=a^3b^2[b(a-c)+c(b-c)]+b^3c^2[c(b-a)+a(c-a)]+c^3a^2[a(c-b)+b(a-b)]$$
$$=(a-b)(a^2bc^3-b^3c^3)+(b-c)(a^3b^2c-a^3c^3)+(a-c)(a^3b^3-ab^3c^2)$$
$$=(a-b).bc^3.(a^2-b^2)+(b-c).a^3c.(b^2-c^2)+(a-c)(.ab^3.(a^2-c^2)$$
$$=(a-b)^2.bc^3(a+b)+(b-c)^2.a^3c(b+c)+(a-c)^2.ab^3(a+c) \geq0$$
Không cần dùng giả thiết $a \geq b \geq c$, tìm mãi mà không thấy sai chỗ nào :ohmy:



#334925 Cho $a\ge b\ge c>0$. Chứng minh rằng : $$...

Đã gửi bởi le_hoang1995 on 12-07-2012 - 19:27 trong Bất đẳng thức và cực trị

Tớ mở đầu nhé, cách hơi trâu bò :lol:
Theo CS ta có
$$\left ( \frac{a^2b}{c^2}+\frac{b^2c}{a^2}+\frac{c^2a}{b^2} \right )\left ( \frac{c^2}{b}+\frac{a^2}{c}+\frac{b^2}{a} \right )\geq (a+b+c)^2$$
Ta sẽ chứng minh $$ \frac{a^2b}{c^2}+\frac{b^2c}{a^2}+\frac{c^2a}{b^2}\geq \frac{c^2}{b}+\frac{a^2}{c}+\frac{b^2}{a} $$
Thật vậy $$ \frac{a^2b}{c^2}+\frac{b^2c}{a^2}+\frac{c^2a}{b^2}- \frac{c^2}{b}+\frac{a^2}{c}+\frac{b^2}{a} $$
$$=\frac{a^2(a-b)(b-c)(ab^2-c^3+b^3-bc^2)+b^4(a^2-c^2)(b-c)+b^2c^2(a^2-b^2)(a-b)}{a^2b^2c^2}\geq 0$$
Suy ra ĐPCM. Dấu bằng xảy ra khi $a=b=c$



#534342 P=$\frac{3x}{y\left ( x+1 \right )} +...

Đã gửi bởi le_hoang1995 on 23-11-2014 - 11:27 trong Bất đẳng thức và cực trị

Bài 1: Cho 2 số dương $x,y$ thỏa mãn $x+y+z=1$. Tìm giá trị lớn nhất:

 

P=$\frac{xy + yz + xz}{x^{2} + y^{2} + z^{2}} + x^{2} + y^{2} +z^{2}$ 

Ta có $x^2+y^2+z^2\geq\frac{(x+y+z)^2}{3}=\frac{1}{3}$ và $x^2+y^2+z^2< (x+y+z)^2=1$

Đặt $x^2+y^2+z^2=a$, ta được $\frac{1}{3}\leq a<1$

Ta có $$P=\frac{xy+yz+zx}{x^2+y^2+z^2}+x^2+y^2+z^2=\frac{(x+y+z)^2-(x^2+y^2+z^2)}{2(x^2+y^2+z^2)}+x^2+y^2+z^2=\frac{1}{2a}-\frac{1}{2}+a$$

Dự đoán Max tại $a=\frac{1}{3}$  khi đó $ P=\frac{4}{3}$ nên ta thử chứng minh

$$a-\frac{1}{2}+\frac{1}{2a}\leq \frac{4}{3}\Leftrightarrow (3a-1)(2a-3)\leq 0\Leftrightarrow \frac{1}{3}\leq a\leq \frac{3}{2}$$

Như vậy với $\frac{1}{3}\leq a<1$ thì $ P\leq \frac{4}{3}$




#299992 Sử dụng phương pháp: Công thức nghiệm của phương trình bậc 2

Đã gửi bởi le_hoang1995 on 19-02-2012 - 15:13 trong Bất đẳng thức và cực trị

Sử dụng phương pháp: Công thức nghiệm của phương trình bậc 2
(lớp 8 chưa học cái này mà cô mình chém bọn mình như đúng rồi, tuy nhiên vẫn post cho anh em nhâm nhi nha)

@@1:

cho 3 số a,b,c dương thỏa mãn các ĐK: a>0, bc=2a, a+b+c=abc
CMR: $a\geq \sqrt{\frac{1+2\sqrt{2}}{2}}$

@@2:
Cho a,b,c thỏa mãn (a+c)(a+b+c)<0. CMR:
$(b-c)^{2} > 4a(a+b+c)$

@@3:
Cho a,b,c thỏa mãn: a+b+c=2 và ab+bc+ca=1 CMR:
$0\leq a,b,c\leq \frac{4}{3}$


BÀI 3.

Ta có $a+b+c=2\Rightarrow b+c=2-a=S$

Và $bc=1-a(b+c)=1-a(2-a)=a^2-2a+1=P$

Như vậy b,c là nghiệm của phương trình $X^2-S*X+P=0$

Cần có $S^2-4P\geq 0$

Thay S,P vào rồi giải ra, ta suy ra điều phải chứng minh.

Bài 1 cũng tương tự :icon6:

$b+c=abc-a=a*2a-a=2a^2-a$ và $bc=2a$



#275396 bất đẳng tưởng đơn giản nhưng khó đấy!

Đã gửi bởi le_hoang1995 on 06-09-2011 - 00:39 trong Bất đẳng thức và cực trị

cho mình hỏi câu này nha, mình làm mãi không ra, dạo này đầu óc hơi bã đậu quá
Khẳng định hoặc phủ định bất đẳng thức sau:
với mọi x;y thuộc R, x và y lớn hơn hoặc bắng 3, x^2 lớn hơn hoặc bắng 3y, CMR:
xy+3 lớn hơn hoặc bằng 2(x+y)



#333549 Tìm GTNN của \[F=\sum\limits_{k=1}^{n-1}{k{x_k}\left({2n-...

Đã gửi bởi le_hoang1995 on 09-07-2012 - 12:02 trong Bất đẳng thức - Cực trị

Anh Thành xem lại em là tìm min hay tìm max vậy! Nếu là tìm max thì em làm như sau:
Ta có:
\[F = \sum\limits_{k = 1}^{n - 1} {k{x_k}\left( {2n - k} \right)} = 2n\sum\limits_{k = 1}^{n - 1} {k{x_k}} - \sum\limits_{k = 1}^{n - 1} {{k^2}{x_k}} = 4n\left( {n - 1} \right) - \sum\limits_{k = 1}^{n - 1} {{k^2}{x_k}} \]
Lại có:
\[\begin{array}{l}
{\left( {\sum\limits_{k = 1}^{n - 1} {{x_k}} } \right)^2} \le \left( {\sum\limits_{k = 1}^{n - 1} {\frac{1}{{{k^2}}}} } \right)\left( {\sum\limits_{k = 1}^{n - 1} {{k^2}{x_k}} } \right) \le \frac{{{\pi ^2}}}{6}\sum\limits_{k = 1}^{n - 1} {{k^2}{x_k}} \\
\Rightarrow \sum\limits_{k = 1}^{n - 1} {{k^2}{x_k}} \ge \frac{{6{n^2}}}{{{\pi ^2}}} \\
\end{array}\]
\[ \Rightarrow F \le 4n\left( {n - 1} \right) - \frac{{6{n^2}}}{{{\pi ^2}}}\]

Cái cauchy-schwarz của bạn, $x_k$ mũ bình chứ (VP)



#336268 $$\sum a_i \sum b_i \ge \sum (a_i+b_i) \su...

Đã gửi bởi le_hoang1995 on 16-07-2012 - 09:00 trong Bất đẳng thức và cực trị

Bài toán
Cho các số thực duơng $a_1,a_2, ..., a_n, b_1, b_2, ..., b_n$. Chứng minh rằng :
$$\left (\sum_{i=1}^n a_i\right )\left (\sum_{i=1}^n b_i\right )\ge \left [\sum_{i=1}^n (a_i+b_i)\right ]\left (\sum_{i=1}^n \dfrac{a_ib_i}{a_i+b_i}\right )$$

Viết lại BĐT như sau
$$\sum \frac{a_k.b_k}{a_k+b_k}\leq \frac{(\sum a_k)(\sum b_k)}{\sum a_k+\sum b_k}$$.
BĐT này có thể chứng minh bằng BĐT cauchy-schwarz.
Chú ý đẳng thức $\frac{a+b}{4}-\frac{ab}{a+b}=\frac{(a-b)^2}{4(a+b)}$.
BĐT tương đương
$$\Leftrightarrow \frac{\sum a_k+\sum b_k}{4}- \frac{(\sum a_k)(\sum b_k)}{\sum a_k+\sum b_k}\leq \sum \left ( \frac{a_k+b_k}{4}-\frac{a_k.b_k}{a_k+b_k} \right )$$
$$\Leftrightarrow \frac{(\sum a_k-\sum b_k)^2}{4(\sum a_k+\sum b_k)}\leq \sum \frac{(a_k-b_k)^2}{4.(a_k+b_k)}$$
$$\Leftrightarrow \frac{[(a_1+a_2+...+a_n)-(b_1+b_2+...+b_n)]^2}{a_1+a_2+...+a_n+b_1+b_2+...+b_n}\leq \frac{(a_1-b_1)^2}{a_1+b_1}+\frac{(a_2-b_2)^2}{a_2+b_2}+...+\frac{(a_n-b_n)^2}{a_n+b_n}$$
Nhưng BĐT này đúng theo BĐT Cauchy-schwarz. ĐPCM



#312867 $$(1+a^{25}b^4c^{2012})\left (\dfrac{1}{1+a^{25}}+\d...

Đã gửi bởi le_hoang1995 on 26-04-2012 - 21:33 trong Bất đẳng thức và cực trị

Nếu đặt $a^{25}=x, b^4=y,c^{2012}=z\Rightarrow x,y,x\in (0,1]$ $\Rightarrow xyz\leq \sqrt[3]{xyz}$

Ta có BĐT sau: với $0\leq x,y\leq 1$ thì $\frac{1}{1+x^2}+\frac{1}{1+y^2}\leq \frac{2}{1+\sqrt{xy}}$

Vì $\frac{1}{1+x^2}+\frac{1}{1+y^2}-\frac{2}{1+\sqrt{xy}}=\frac{(x-y)^2(xy-1)}{(1+x^2)(1+y^2)(1+\sqrt{xy})}\leq 0$

Sử dụng BĐT mình đã chứng minh ở trên, ta có

$\frac{1}{1+a^3}+\frac{1}{1+b^3}\leq \frac{2}{1+\sqrt{(ab)^3}}$

$\frac{1}{1+c^3}+\frac{1}{1+abc}\leq \frac{2}{1+\sqrt{abc^4}}$

Cộng theo vế, ta được

$\frac{1}{1+a^3}+\frac{1}{1+b^3}+\frac{1}{1+c^3}+\frac{1}{1+abc}\leq 2\left ( \frac{1}{1+\sqrt{(ab)^3}}+\frac{1}{1+\sqrt{abc^4}} \right )\leq \frac{4}{1+abc}$

$\Leftrightarrow \frac{1}{1+a^3}+\frac{1}{1+b^3}+\frac{1}{1+c^3}\leq \frac{3}{1+abc}$


Như vậy ta có

$f(x,y,z)=(1+xyz)\left ( \frac{1}{1+x}+\frac{1}{1+y}+\frac{1}{1+z} \right )\leq (1+\sqrt[3]{xyz}).\frac{3}{1+\sqrt[3]{xyz}}=3$



#299784 Chứng minh : $ a^3+b^3+c^3+3abc \ge ab\sqrt{2(a^2+b^2)}+bc...

Đã gửi bởi le_hoang1995 on 17-02-2012 - 20:42 trong Bất đẳng thức và cực trị

Đây là làm mạnh BĐT schur, mình không làm được nhưng tìm được nó bên Mathscope.

http://forum.mathsco...thread.php?t=87



#346964 BẤT ĐẲNG THỨC GIÁ TRỊ TƯƠNG ĐƯƠNG

Đã gửi bởi le_hoang1995 on 15-08-2012 - 17:13 trong Bất đẳng thức - Cực trị

Cho dãy $(c_{n})$ dương.CM
$\prod_{k=1}^{n}(1+c_{k})\leq (1+\frac{\sum_{k=1}^{n}c_{n}}{n})^{n}$

Xét hàm số $f(x)=ln(1+x)$ với x dương $\Rightarrow f'(x)=\frac{1}{1+x}\Rightarrow f''(x)=\frac{-1}{(1+x)^2}<0$
Suy ra hàm số lõm. Áp dụng BĐT Jensen ta được:
$$f(c_1)+f(c_2)+...+f(c_n)\leq nf(\frac{c_1+c_2+...+c_n}{n})$$
$$\Leftrightarrow ln(1+c_1)+ln(1+c_2)+...+ln(1+c_n)\leq n.ln\left ( 1+\frac{c_1+c_2+...+c_n}{n} \right )$$
$$\Leftrightarrow (1+c_1)(1+c_2)...(1+c_n)\leq \left ( 1+\frac{\sum_{k=1}^{n}c_n}{n} \right )^n$$



#332024 $\dfrac{1}{a}+\dfrac{1}{b}+\dfrac{1}{c}+a+b+c \geq 4...

Đã gửi bởi le_hoang1995 on 05-07-2012 - 01:46 trong Bất đẳng thức và cực trị

cho a,b,c>0 TM $a^2+b^2+c^2=1$ CMR:

$\dfrac{1}{a}+\dfrac{1}{b}+\dfrac{1}{c}+a+b+c \geq 4\sqrt{3}$

Một cách khác, theo CS ta có $\sqrt{3(a^2+b^2+c^2)}\geq a+b+c\Rightarrow a+b+c\leq \sqrt{3}$.
Áp dụng BĐT AM-GM ta được
$$VT\geq \frac{9}{a+b+c}+(a+b+c)=\frac{6}{a+b+c}+\left [ \frac{3}{a+b+c}+(a+b+c) \right ]\geq \frac{6}{\sqrt{3}}+2\sqrt{3}=4\sqrt{3}$$
Dấu bằng xảy ra khi $a=b=c=\frac{1}{\sqrt{3}}$